How to Solve a Double Summation Problem?

  • Thread starter Thread starter mexion
  • Start date Start date
  • Tags Tags
    Sigma Sum
Click For Summary
The double summation problem involves calculating the expression ∑_{k=1}^{3} ∑_{j=0}^{4} k^{j}, which simplifies to 157 when evaluated correctly. The discussion highlights that the order of summation does not affect the final result, as demonstrated using MATLAB. Another summation problem, ∑_{k=0}^{4} ∑_{j=1}^{5} (3^{k} + jk), is addressed, with the suggestion to start with the sum over j to simplify calculations. The final result for this second summation is indicated to be 20 after evaluating the terms. Understanding the properties of summation can streamline solving these types of problems without software assistance.
mexion
Messages
5
Reaction score
0
Homework Statement
\sum_{k=1}^{3} \sum_{j=0}^{4}k^{j}

The attempt at a solution
so this above means:
(1+1^{1}+1^{2}+1^{3}+1^{4})*(1+2^{1}+2^{2}...)*(1+3...)*...
or
(1*1^{1}*1^{2}*1^{3}*1^{4})+(1*2^{1}*2^{2}...)+(1*3...)+...
 
Physics news on Phys.org
It doesn't make a difference which order you do it in. You can see if you type the following into matlab:

syms k
syms j

symsum(symsum(k^j,j,0,4),k,1,3)

or,

symsum(symsum(k^j,k,1,3),j,0,4)

the answer is 157 either way.
 
so 1 more question
\sum_{k=0}^{4} \sum_{j=1}^{5} (3^{k} + jk)
how to calculate this without Matlab.
Should I transform it? (but have no idea how)
or just have to put [0;4] for k, and [1;5] for j, and add everything ?
 
You don't need to do a transform, if you start with the sum over j you will get

\sum_{k=0}^{4} (3^k+1k+ ... 3^k+5k) = 0+ ...+ 3^4+20
 
mexion said:
Homework Statement
\sum_{k=1}^{3} \sum_{j=0}^{4}k^{j}

The attempt at a solution
so this above means:
(1+1^{1}+1^{2}+1^{3}+1^{4})*(1+2^{1}+2^{2}...)*(1+3...)*...
or
(1*1^{1}*1^{2}*1^{3}*1^{4})+(1*2^{1}*2^{2}...)+(1*3...)+...

It means the second one (which = 157); the first would be product_{k=1..3} sum_{j=0..4} k^j = 18755.

RGV
 
Question: A clock's minute hand has length 4 and its hour hand has length 3. What is the distance between the tips at the moment when it is increasing most rapidly?(Putnam Exam Question) Answer: Making assumption that both the hands moves at constant angular velocities, the answer is ## \sqrt{7} .## But don't you think this assumption is somewhat doubtful and wrong?

Similar threads

  • · Replies 3 ·
Replies
3
Views
1K
  • · Replies 3 ·
Replies
3
Views
2K
  • · Replies 2 ·
Replies
2
Views
1K
  • · Replies 14 ·
Replies
14
Views
2K
  • · Replies 1 ·
Replies
1
Views
1K
  • · Replies 22 ·
Replies
22
Views
909
Replies
9
Views
2K
  • · Replies 7 ·
Replies
7
Views
2K
  • · Replies 6 ·
Replies
6
Views
1K
  • · Replies 2 ·
Replies
2
Views
1K